todavidzheng
Thanks Received: 0
Forum Guests
 
Posts: 12
Joined: January 09th, 2011
 
 
 

Q20 - Advertisement: Each of the Economic

by todavidzheng Thu Jan 27, 2011 1:16 pm

Hello,
Could anyone please elaborate on why B is not correct? and why D is the right answer? I am having a hard time to choose between this two...
Thank you!
 
itzakadoozie412
Thanks Received: 0
Vinny Gambini
Vinny Gambini
 
Posts: 17
Joined: September 13th, 2010
 
 
 

Re: Q20 - Advertisement: Each of the Economic

by itzakadoozie412 Sun Feb 06, 2011 2:22 pm

Same here! Could I also get an explanation of picking (D) over (B). I need a good reason to eliminate (B).
User avatar
 
ManhattanPrepLSAT1
Thanks Received: 1909
Atticus Finch
Atticus Finch
 
Posts: 2851
Joined: October 07th, 2009
 
This post thanked 5 times.
 
 

Re: PT44, S4, Q20 - Advertisement: Each of the

by ManhattanPrepLSAT1 Sun Feb 06, 2011 4:40 pm

This is a great question! Thanks for bringing it to the forum...

Let's look at the argument core first. There is a premise that the Merit Prize winners are covered by the Acme retirement plan. There is a intermediate conclusion that the Merit Prize winners have recognized that the Acme plan offers a financially secure future. And then the argument concludes that the plan is probably good for anyone with similar retirement needs.

There are several assumptions in here but the one most glaring to me was that the intermediate conclusion does not follow from the premise. Many people have their retirement plans chosen for them by their employer, so the possession of a retirement plan does not translate to its endorsement. This issue is pointed out in answer choice (D).

Let's look at the answer choices:

(A) is out of scope. The argument never claims that the Acme plan is the only good plan so who cares if there was another plan used previously.
(B) is out of scope for the same reason as answer choice (A). The argument doesn't claim that the Acme plan is the only or even the best plan available.
(C) is close too, but the flaw is really between the premise and the intermediate conclusion. This answer choice implies that it's the main conclusion that contains the flaw.
(E) is not true. The argument only concludes finally that the plan would be good for anyone with similar needs, not for anyone in general.

Hope that helps!
 
zainrizvi
Thanks Received: 16
Atticus Finch
Atticus Finch
 
Posts: 171
Joined: July 19th, 2011
 
 
trophy
First Responder
 

Re: Q20 - Advertisement: Each of the

by zainrizvi Wed Sep 28, 2011 11:21 am

Does (C) fail as a choice since these aren't "supposed experts" but really are experts relevant to the topic at hand? Is this the reason why this appeal to authority is legitimate?
 
americano1990
Thanks Received: 25
Jackie Chiles
Jackie Chiles
 
Posts: 29
Joined: April 24th, 2011
 
This post thanked 4 times.
 
 

Re: Q20 - Advertisement: Each of the

by americano1990 Mon Nov 07, 2011 9:06 pm

I am not sure if relying on the term "supposed experts" is the safest reason to get rid of (C). Granted, the argument does not give any specifications regarding the identity of the "supposed experts", but I also do not think it is brutally wrong to loosen up the language here a bit and presume for now that supposed experts are the winners of the prize...

So as for your question, "Is this the reason why this appeal to authority is legitimate?" all i can say is that it is not definitely illegitimate. But also to say it IS legitimate...would be again difficult to back up. The legitimacy of the appeal to authority is not the main isssue here.

Anyways the surest way to get rid of (C) (even if you loosen up the language as i said above) would be to look at the phrase "it appeals to the fact that supposed experts have ENDORSED the argument's main conclusion." It DOESNT appeal to their ENDORSEMENT. The advertisement is simply using the fact that these planners are USING this retirement plan and extrapolates from that fact that these people have RECOGNIZED its value. But nowhere do we get any specific reference to--let alone appeal to-- the ENDORSEMENT of the argument's conclusion by these experts PER SE.

So in the end, the issue with "supposed experts" is insignficant here. Whether or not it would constitute a legitimate appeal to authority, there is another huge flaw in the answer choice (C) that allows us to get rid of it.
 
gmatalongthewatchtower
Thanks Received: 1
Jackie Chiles
Jackie Chiles
 
Posts: 47
Joined: November 22nd, 2011
 
This post thanked 1 time.
 
 

Re: Q20 - Advertisement: Each of the

by gmatalongthewatchtower Mon Jan 30, 2012 12:05 pm

Manhattan LSAT Instructors,

I have a question about D. First of all, I was able to recognize the scope shift from "allotment" of plans to "endorsement" of plans. However, the correct answer choice says that only *some* winners have endorsed the plan. This is nowhere assumed or stated in the prompt. The prompt says that "the winners ... have clearly recognized ...." Hence, the prompt seems to be talking about *all* winners not just a subset of winners. Isn't it?

Secondly, I chose C because it says that the argument appeals to fact that supposed experts (or winners IMO) have endorsed the main conclusion. Isn't that true? Winners have not endorsed the viability of the plan. There is no evidence for it.

I need your guidance. Can you please help me? :(

Thanks
GMATAlongTheWatchtower
 
timmydoeslsat
Thanks Received: 887
Atticus Finch
Atticus Finch
 
Posts: 1136
Joined: June 20th, 2011
 
This post thanked 2 times.
 
trophy
Most Thanked
trophy
First Responder
 

Re: Q20 - Advertisement: Each of the

by timmydoeslsat Mon Jan 30, 2012 7:41 pm

gmatalongthewatchtower Wrote:Manhattan LSAT Instructors,

I have a question about D. First of all, I was able to recognize the scope shift from "allotment" of plans to "endorsement" of plans. However, the correct answer choice says that only *some* winners have endorsed the plan. This is nowhere assumed or stated in the prompt. The prompt says that "the winners ... have clearly recognized ...." Hence, the prompt seems to be talking about *all* winners not just a subset of winners. Isn't it?

Secondly, I chose C because it says that the argument appeals to fact that supposed experts (or winners IMO) have endorsed the main conclusion. Isn't that true? Winners have not endorsed the viability of the plan. There is no evidence for it.

I need your guidance. Can you please help me? :(

Thanks
GMATAlongTheWatchtower


I am going to help you develop a mental process. This test is all about asking the right questions to yourself and spotting the conclusion along with the evidence that supports the claim.

This is an identity the flaw question. So we know that we are dealing with a flawed argument.

We are to take the premises as fact. What we can argue with are the conclusions drawn from those facts. That is what is at issue here and almost every argument given.

The stimulus in a nutshell:

Each of the EMP winners from the past 25 years is covered by the ACME retirement plan.

We know this for a fact.

The argument goes on to conclude from this statement that the winners have recognized that this plan offers them a financially secure future.

The argument then concludes from this intermediate conclusion that it is probably a good plan for anyone with retirement needs similar to theirs.

So we have this structure:

(1)Premise ---> (2)Intermediate Conclusion ---> (3)Main conclusion

(1) Each of the EMP winners of the last 25 years have the ACME retirement plan.

(2) Therefore, the winners have recognized that the retirement plan offers them a financially secure future.

(3) It is probably a good plan for anyone with retirement needs similar to theirs.

We can take issue with statements 2 and 3. The flaw is clearly the fact that we do not even know if these winners picked these retirement plans! To go from statement (1) to statement (2) is a huge leap isn't it?

Answer choices:

A) We are making conclusions based on those past 25 year winners. We are not making judgments involving possible previous winners.

B) It does not prohibit the possibility that there other plans that can offer a financially safe future. It simply statements that this is one.

C) The winners have not endorsed the main conclusion! They have not commented at all about the probability of it being a good plan for others in similar needs. All we know about these winners is that they have the ACME retirement plan.

E) The author is not assuming that each of the winners has identical needs. The conclusion is "probably"...so we already have wiggle room there. We have no reason to believe that the author is assuming that EACH of the winners do anything. Plus, the stimulus stated similar needs, not identical.
 
gmatalongthewatchtower
Thanks Received: 1
Jackie Chiles
Jackie Chiles
 
Posts: 47
Joined: November 22nd, 2011
 
 
 

Re: Q20 - Advertisement: Each of the

by gmatalongthewatchtower Mon Feb 27, 2012 2:55 pm

Timmy - Thanks for your detailed reply. Appreciate it. I could understand your logic. However, my original question about "Some" and "Each of EMP winners" is still unanswered. :(

The reason why I didn't choose Ans. Choice D is that it talks about a subset of people not electing to choose the retirement plan. The prompt says that "each of the winners" is covered.

Can you please help me? :(
 
timmydoeslsat
Thanks Received: 887
Atticus Finch
Atticus Finch
 
Posts: 1136
Joined: June 20th, 2011
 
 
trophy
Most Thanked
trophy
First Responder
 

Re: Q20 - Advertisement: Each of the

by timmydoeslsat Mon Feb 27, 2012 3:10 pm

gmatalongthewatchtower Wrote:Timmy - Thanks for your detailed reply. Appreciate it. I could understand your logic. However, my original question about "Some" and "Each of EMP winners" is still unanswered. :(

The reason why I didn't choose Ans. Choice D is that it talks about a subset of people not electing to choose the retirement plan. The prompt says that "each of the winners" is covered.

Can you please help me? :(


What answer choice D sets up is that the argument is assuming something, which is a flaw.

"It takes for granted that..." = necessary assumption. We can use the negate test to see if the argument really did assume that.

And in this case, it does. I will negate choice D.

No winners have deliberately selected their plan rather than having it chosen for them.

The word some is just really soft language that could imply all of course. But the negation of some = none, and in which case, we can see that some is necessary for this argument to survive. Since it was not stated, it is an assumption. And since it is an assumption, it is a flaw in the argument.
 
aprilhu33
Thanks Received: 0
Vinny Gambini
Vinny Gambini
 
Posts: 5
Joined: June 24th, 2012
 
 
 

Re: Q20 - Advertisement: Each of the Economic

by aprilhu33 Mon Aug 26, 2013 4:04 am

It was previously stated that all premises are facts and can't be argued. But how do you know sure that the endorsement of ACME is not a premise but an intermediate conclusion? Basically, I chose C b/c I took the endorsement part as a undisputed fact, a premise, but I was wrong. How would I avoid this in the future?
 
foralexpark
Thanks Received: 2
Vinny Gambini
Vinny Gambini
 
Posts: 24
Joined: June 08th, 2013
 
 
trophy
First Responder
 

Re: Q20 - Advertisement: Each of the Economic

by foralexpark Mon Oct 14, 2013 6:37 pm

aprilhu33 Wrote:It was previously stated that all premises are facts and can't be argued. But how do you know sure that the endorsement of ACME is not a premise but an intermediate conclusion? Basically, I chose C b/c I took the endorsement part as a undisputed fact, a premise, but I was wrong. How would I avoid this in the future?



I missed this question for the same problem you mentioned.

Looking at it again, there is conclusion indicator:

"Since the winners of the nation's most prestigious award for economists have THUS CLEARLY recognized that the acme plan offers them a financially secure future..."

that phrase THUS CLEARLY is what distinguishes premise vs. conclusion...

and also, the source of this argumentation is from an Advertisement, so it looks like it's making an unwarranted argument (i know this is prolly not a good explanation)
User avatar
 
ManhattanPrepLSAT1
Thanks Received: 1909
Atticus Finch
Atticus Finch
 
Posts: 2851
Joined: October 07th, 2009
 
 
 

Re: Q20 - Advertisement: Each of the Economic

by ManhattanPrepLSAT1 Mon Oct 14, 2013 7:34 pm

That's a great explanation foralexpark!

In this argument we can identify the intermediate conclusion by using the structural indicator as you suggested.

The main argument is organized around the word "since," while the intermediate argument can be seen by the word "thus" contained with the evidence offered in support of the argument's main conclusion.

Nice work foralexpark!
 
lsatzen
Thanks Received: 0
Jackie Chiles
Jackie Chiles
 
Posts: 27
Joined: February 25th, 2014
 
 
 

Re: Q20 - Advertisement: Each of the Economic

by lsatzen Tue Apr 15, 2014 8:29 pm

Hello MLSAT Instructors!

I am having a bit of trouble with understanding the reasoning behind this question and would love some clarification.

Here is how I broke down the stimulus and my thought processes for answering the question:

Premise(s):
(1) Each of the EMP winners from the past 25 years covered by Acme retirement plan (fact)
(2) winners of the award clearly recognized it offers them security

Conclusion:
(1) It is probably a good plan for anyone with similar retirement needs.

In my first run in with this question, I had several pre-phrases going on in my head prior to jumping into the questions. Broadly speaking, the correct AC for Flaw questions will either 1) highlight an implicit / underlying assumption which the argument relies on or 2) brings to light a neglected alternative that the author has failed to consider. With this in mind, I tossed around the idea that the author makes an unwarranted assumption that the terms "financially secure future" and "good plan" mean the same thing, when in fact they could mean different things to different people (what if, for some odd reason, people enjoy financial INsecurity because they are ignorant of proper financial management or they just plain dislike financial security). I was also kicking around the idea that the argument is committing the appeal to authority fallacy (which turns out to be incorrect).

Answer Choices:
A) Eliminated as out-of-scope. The core of the argument is concerned with the relation between those who have used the Acme plan and those with similar needs of the users of the Acme plan.
B) Eliminated as out-of-scope. The argument does not attempt to establish that people with similar needs must use the Acme plan, or that it is the only plan for them, rather it merely states that it probably is a good plan for them. So considering the possibility that several other plans would be good enough does not portray a flaw in the argumentation.
C) Initially chose this AC, but realized that it was wrong during blind review. The reason I eliminated it was because it states that the supposed experts have endorsed the argument's main conclusion, when in fact they might not. The main conclusion was "it is probably a good plan...", but the unidentified experts (which I am guessing are the prize winners) only recognize that it offers them a financially secure future (the intermediate conclusion).
D) I chose D through process of elimination, but still do not understand why it is the correct answer choice. Matt Sherman's analysis of the question hones in on the disconnect between possession and endorsement, but I thought that this was a bit of a stretch given the amount of mental leg-work one must do. Under MS's analysis of the assumption, the author of the argument takes for granted that being in possession of something implies the endorsement of the that thing - which does not have to be the case. This proves problematic for the author's argument because it showcases the possibility that the EMP winners might not recognize it as offering them a financially secure future, maybe they were forced to accept it and are unsure of its future financial benefits. If its possible that they no longer recognize it as offering them the purported benefits, then the author can no longer use their recognition as support for the fact that it is a good plan for others with similar needs.

But my initial thoughts in response to answer choice D were: "so what if they didn't deliberately choose the retirement plan, isn't it also possible that although they did not choose the plan, they still recognize it as offering them a financially secure future?" Is the one side of the possibilities enough to count as grounds for vulnerability? Is our task in flaw questions just to cast the slightest bit of doubt over the argumentation?
E) Eliminated. Inconsistent with the stimulus.

Any insight would be deeply appreciated!
User avatar
 
maryadkins
Thanks Received: 640
Atticus Finch
Atticus Finch
 
Posts: 1261
Joined: March 23rd, 2011
 
 
 

Re: Q20 - Advertisement: Each of the Economic

by maryadkins Thu Apr 17, 2014 4:18 pm

Thanks for your questions, Glen.

glen_oh Wrote:D) I chose D through process of elimination, but still do not understand why it is the correct answer choice. Matt Sherman's analysis of the question hones in on the disconnect between possession and endorsement, but I thought that this was a bit of a stretch given the amount of mental leg-work one must do. Under MS's analysis of the assumption, the author of the argument takes for granted that being in possession of something implies the endorsement of the that thing - which does not have to be the case. This proves problematic for the author's argument because it showcases the possibility that the EMP winners might not recognize it as offering them a financially secure future, maybe they were forced to accept it and are unsure of its future financial benefits. If its possible that they no longer recognize it as offering them the purported benefits, then the author can no longer use their recognition as support for the fact that it is a good plan for others with similar needs.


This is where you're off. It's actually not much of a stretch. Just because they have a retirement plan definitely doesn't mean they chose it or endorse it. How many people do you know who hate the insurance they're provided through their employers?

glen_oh Wrote:But my initial thoughts in response to answer choice D were: "so what if they didn't deliberately choose the retirement plan, isn't it also possible that although they did not choose the plan, they still recognize it as offering them a financially secure future?" Is the one side of the possibilities enough to count as grounds for vulnerability? Is our task in flaw questions just to cast the slightest bit of doubt over the argumentation?


I suppose it's still possible, but then this argument is meaningless; the entire conclusion is premised on the belief that these people like this plan and think it's good, which is interpreted from their having the plan. We can infer that he assumes they chose it. Otherwise, his argument makes no sense. It's an inference, yes, but a common sense one.

Hope this clarifies!
 
dukeag
Thanks Received: 2
Vinny Gambini
Vinny Gambini
 
Posts: 17
Joined: April 22nd, 2014
 
 
 

Re: Q20 - Advertisement: Each of the

by dukeag Sat May 24, 2014 5:58 pm

These sneaky questions always get me towards the end of the test, especially because I read more quickly and so don't get all the details or the entire picture.

But yeah, I think you are totally right. I picked C too, but now that I look back at the passage, I realize that nowhere does it say that the prize winners personally endorsed this retirement plan. The advertisement simply states this assumption as it if were a fact, and I think because the passage represented this assumption as if it were a fact, I misunderstood it to be a fact. And that's where it GOT ME!

The placement of the "thus" in the sentence was what tricked me too. For some reason I forgot how to read English for a second and misunderstood it to mean "Because the winners recognized the Acme plan to be a good one, THUS it is a good one." But what THUS is really referring to or the fact it is following up on is the fact stated in the previous sentence, which says that prize winners from the past 25 years have been covered by the retirement plan, THUS the prize winners have recognized it is a good plan.

But of course that is an assumption. Just because the prize winners are covered by the plan does not mean they have recognized that it offers them a financially secure future. Maybe they didn't have a choice in picking the retirement plan (D) and maybe they wanted to pick a different plan but were prohibited as a result of the terms receiving the economic merit prize...etc...

americano1990 Wrote:I am not sure if relying on the term "supposed experts" is the safest reason to get rid of (C). Granted, the argument does not give any specifications regarding the identity of the "supposed experts", but I also do not think it is brutally wrong to loosen up the language here a bit and presume for now that supposed experts are the winners of the prize...

So as for your question, "Is this the reason why this appeal to authority is legitimate?" all i can say is that it is not definitely illegitimate. But also to say it IS legitimate...would be again difficult to back up. The legitimacy of the appeal to authority is not the main isssue here.

Anyways the surest way to get rid of (C) (even if you loosen up the language as i said above) would be to look at the phrase "it appeals to the fact that supposed experts have ENDORSED the argument's main conclusion." It DOESNT appeal to their ENDORSEMENT. The advertisement is simply using the fact that these planners are USING this retirement plan and extrapolates from that fact that these people have RECOGNIZED its value. But nowhere do we get any specific reference to--let alone appeal to-- the ENDORSEMENT of the argument's conclusion by these experts PER SE.

So in the end, the issue with "supposed experts" is insignficant here. Whether or not it would constitute a legitimate appeal to authority, there is another huge flaw in the answer choice (C) that allows us to get rid of it.
 
WesleyC316
Thanks Received: 3
Jackie Chiles
Jackie Chiles
 
Posts: 40
Joined: March 19th, 2018
Location: Shanghai
 
 
 

Re: Q20 - Advertisement: Each of the Economic

by WesleyC316 Fri Apr 13, 2018 9:39 am

D is the best among the five answer choices, but I still think D is too, I don't know, arbitrary to me. I don't see any sign in the stimulus that the argument takes that for granted. I mean, even if EVERY winner got enrolled in the plan by their employers, they can still "recognize" the benefits of the plan, don't they? D sounds to me as if not deliberately choosing the plan implies not endorsing the plan. D does make the argument vulnerable the most among the five answers, but I don't think it makes the argument vulnerable enough. Just a thought!